Next Article in Journal
Clausen’s Series 3F2(1) with Integral Parameter Differences
Previous Article in Journal
On Eigenfunctions and Eigenvalues of a Nonlocal Laplace Operator with Multiple Involution
 
 
Font Type:
Arial Georgia Verdana
Font Size:
Aa Aa Aa
Line Spacing:
Column Width:
Background:
Article

The Inequalities of Merris and Foregger for Permanents

by
Divya K. Udayan
*,† and
Kanagasabapathi Somasundaram
Department of Mathematics, Amrita School of Engineering, Amrita Vishwavidyapeetham, Coimbatore 641112, India
*
Author to whom correspondence should be addressed.
These authors contributed equally to this work.
Symmetry 2021, 13(10), 1782; https://doi.org/10.3390/sym13101782
Submission received: 12 August 2021 / Revised: 16 September 2021 / Accepted: 18 September 2021 / Published: 25 September 2021

Abstract

:
Conjectures on permanents are well-known unsettled conjectures in linear algebra. Let A be an n × n matrix and S n be the symmetric group on n element set. The permanent of A is defined as per A = σ S n i = 1 n a i σ ( i ) . The Merris conjectured that for all n × n doubly stochastic matrices (denoted by Ω n ), n per A min 1 i n j = 1 n per A ( j | i ) , where A ( j | i ) denotes the matrix obtained from A by deleting the jth row and ith column. Foregger raised a question whether per ( t J n + ( 1 t ) A ) per A for 0 t n n 1 and for all A Ω n , where J n is a doubly stochastic matrix with each entry 1 n . The Merris conjecture is one of the well-known conjectures on permanents. This conjecture is still open for n 4 . In this paper, we prove the Merris inequality for some classes of matrices. We use the sub permanent inequalities to prove our results. Foregger’s inequality is also one of the well-known inequalities on permanents, and it is not yet proved for n 5 . Using the concepts of elementary symmetric function and subpermanents, we prove the Foregger’s inequality for n = 5 in [0.25, 0.6248]. Let σ k ( A ) be the sum of all subpermanents of order k. Holens and Dokovic proposed a conjecture (Holen–Dokovic conjecture), which states that if A Ω n , A J n and k is an integer, 1 k n , then σ k ( A ) ( n k + 1 ) 2 n k σ k 1 ( A ) . In this paper, we disprove the conjecture for n = k = 4 .

1. Introduction

Let S n be the symmetric group on n element set and let A be an n × n matrix. The permanent of A is defined as
per A = σ S n i = 1 n a i σ ( i ) .
A matrix A is said to be doubly stochastic if it is a real non-negative matrix with each row sum and column sum equal to 1. Let Ω n denote the set of all n × n doubly stochastic matrices. For positive integers n and k with 1 k n , Q k , n denotes the set { i 1 , , i k / 1 i 1 < < i k n } . For α , β Q k , n , let A α / β be the submatrix of A obtained by deleting the rows indexed by α and columns indexed by β and A [ α / β ] be the submatrix of A with rows and columns indexed by α and β , respectively.
For 1 k n , the kth order subpermanent of A is defined by σ k ( A ) = α , β Q k , n per A [ α / β ] . In this paper, we use the following results quoted by Minc [1]: If A and B are two n × n matrices and 1 k n , then
per A = β Q k , n per A [ α / β ] per A α / β , for α Q k , n ,
and
per ( A + B ) = k = 0 n S k A , B ,
where S k A , B = α , β Q k , n per A [ α / β ] per B α / β , per A [ α / β ] = 1 when k = 0 and per B ( α / β ) = 1 when k = n .
Elliott H. Lieb [2] gave proofs of some conjectures on permanents. S G Hwang [3] proved that J n = ( 1 n ) n × n is the unique ϕ -maximizing matrix on K n . Lih and Wang [4] proved the monotonicity conjecture for n = 3 . A survey on conjectures on permanents are given in [5,6].
Merris [7] conjectured that if A Ω n then n per A min 1 i n j = 1 n per A ( j | i ) . He also suggested a method to prove this conjecture. The conjecture is still open for n 4 . Subramanian and Somasundaram [8] have proved that if A Ω n and the polynomial r = 2 n r ( n r ) ! n n r σ r ( A J n ) t r 2 has no root in ( 0 , 1 ) then A satisfies Merris conjecture. Furthermore, they proved some sufficient conditions for matrices in Γ k n to satisfy the Merris conjecture, where Γ k n denote the set of n × n non-negative matrices with each row sum and column sum equal to k.
In Section 2, we prove the Merris inequality for all n × n non-negative matrices with minimum entry greater than or equal to 1 n . We prove that if A is an n × n non-negative matrix with minimum entry greater than or equal to 1 n and maximum entry less than or equal to 1, then n 2 p e r A λ i , where λ i s are the eigenvalues of [ a i j p e r A ( i | j ) ] . Furthermore, we give a sufficient condition for a doubly stochastic matrix A to satisfy the Merris conjecture.
Foregger [9] raised a question whether per ( t J n + ( 1 t ) S ) per S for 0 t n n 1 , and S Ω n . He proved in [9] that for n = 3 , per ( t J 3 + ( 1 t ) S per S for 0 t 3 2 for S Ω 3 with equality iff S = J 3 or t = 3 2 and S is (up to permutations of rows and columns) 1 2 ( I + P ) , where P is a full-cycle permutation matrix. In addition, he proved in [10] that if S Ω 4 has all its off-diagonal entries less than or equal to 9 20 and t 0 < t 4 3 , where t 0 is the unique real root of 106 t 3 418 t 2 + 465 t 100 then per ( t J 4 + ( 1 t ) S ) per S with equality if and only if S = J 4 .
Subramanian and Somasundaram [8] proved that if A Ω n , 2 k n , and the polynomial r = 2 k r c r σ r ( A J n ) t r 2 has no root in ( 0 , 1 ) , where c r = ( k r ) ! n k r n r k r 2 , then σ k ( t A + ( 1 t ) J n ) σ k ( A ) for all t [ 0 , 1 ] . In Section 3, we prove that for all S Ω 5 and all t such that 0.25 t 0.6248 , per ( t J 5 + ( 1 t ) S ) per S .
Holens [11] and Dokovic [12] proposed a conjecture (Holen–Dokovic conjecture), which states that if A Ω n , A J n and k is an integer, 1 k n , then σ k ( A ) ( n k + 1 ) 2 n k σ k 1 ( A ) . S G Hwang [13] proved the conjecture for an n 2 dimensional face of Ω n . Wanless [14] disproved this conjecture by providing a counterexample of order 22. The smallest order of a counterexample has not been established. In Section 3, we prove that the Holen–Dokovic conjecture fails for n = k = 4 and thus established that the smallest order of a counterexample to Holen–Dokovic conjecture is 4.

2. Merris Conjecture

Let Γ k n denote the set of n × n non-negative matrices with each row sum and column sum equal to k. Merris [7] conjectured that for all n × n doubly stochastic matrices,
n per A min 1 i n j = 1 n per A ( j | i ) .
He also raised a question whether
n per A max 1 i n j = 1 n per A ( j | i ) for all A Ω n .
The Merris conjecture is one of the well-known conjectures in linear algebra, in particular on permanent. The conjecture is still open for n 4 . There is not much progress in this conjecture. Subramanian and Somasundaram [8] have proved that if A Ω n and the polynomial r = 2 n r ( n r ) ! n n r σ r ( A J n ) t r 2 has no root in ( 0 , 1 ) then A satisfies the Merris conjecture, and they also proved some sufficient conditions for matrices in Γ k n to satisfy the Merris conjecture.
A matrix is said to be a positive matrix if all its entries are non-negative [15]. Let A i be k × k matrix, i = 1 , 2 , , n . The direct sum of the matrices A i is defined as follows:
i = 1 n A i = diag ( A 1 , A 2 , , A n ) = A 1 0 0 0 A 2 0 0 0 A n , where 0 is the zero matrix.
Lemma 1.
If A is a n × n positive matrix with minimum entry greater than or equal to 1 n , then
  • n per A max 1 i n j = 1 n per A ( j | i ) .
  • n per A min 1 i n j = 1 n per A ( j | i ) .
  • n 2 per A i , j = 1 n per A ( i | j ) .
Proof. 
  • We need to find max 1 i n j = 1 n per A ( j | i ) . Let the maximum sum be attained in the kth column, i.e., max 1 i n j = 1 n per A ( j | i ) = j = 1 n per A ( j | k ) , where 1 k n . Let the entries of the kth column be k 1 , k 2 , , k n . This implies that 1 n k l for each l = 1 , 2 , , n and hence 1 n k l for each l = 1 , 2 , , n . Taking the permanent along the kth column, per A = i = 1 n k i per ( i | k ) . Multiplying by n on both sides, n per A = i = i n n k i per A ( i | k ) . Since n k l 1 for each l = 1 , 2 , , n and since each of the subpermanents is non-negative, this implies that n per A i = 1 n per A ( i | k ) . This implies that n per A max 1 i n j = 1 n per A ( j | i ) .
  • From the inequality 1, n per A min 1 i n j = 1 n per A ( j | i ) .
  • From the proof of the inequality 1, n per A j = 1 n per A ( j | i ) for each i = 1 , 2 , , n . Taking summation over i , i running from 1 to n, n 2 per A i = 1 n j = 1 n per A ( j | i ) . n 2 per A i , j = 1 n per A ( i | j ) .
Theorem 1.
If A is a n × n positive matrix with constant columns and maximum entry greater than or equal to 1 n then A satisfies the inequality
n p e r A min 1 i n j = 1 n p e r A ( j | i ) .
Proof. 
Suppose A = ( a i j ) = k j , for all i. Then per A = n ! k 1 k 2 k n .   j = 1 n per A ( j | i ) = n ( n 1 ) ! k 1 k 2 k i 1 k i + 1 k n = n ! k 1 k 2 k i 1 k i + 1 k n   = per A k i . min 1 i n j = 1 n per A ( j | i ) = per A k l , where k l = max { k 1 , k 2 , k n } . Since, k l 1 n , we have n per A min 1 i n j = 1 n p e r A ( j | i ) per A k l min 1 i n j = 1 n per A ( j | i ) = 0 . Therefore, n per A min 1 i n j = 1 n per A ( j | i ) .
Theorem 2.
If A is a n × n matrix whose minimum entry is greater than or equal to 1 n and maximum entry is less than or equal to 1 then n 2 per A λ i , where λ i is an eigenvalue of [ a i j per A ( i | j ) ] .
Proof. 
If A is an n × n non-negative matrix whose minimum entry is greater than or equal to 1 n then from Lemma 1, n 2 per A i , j = 1 n per A ( i | j ) .
per A 1 n 2 i , j = 1 n per A ( i | j ) .
Let a l m be the maximum entry of A. Multiplying on both sides by a l m ,
a l m per A 1 n 2 i , j = 1 n a l m per A ( i | j ) 1 n 2 i , j = 1 n a i j per A ( i | j ) .
per A 1 n 2 a l m i , j = 1 n a i j per A ( i | j ) .
By the assumption a l m 1 , 1 a l m 1 .
per A 1 n 2 i , j = 1 n a i j per A ( i | j ) 1 n 2 i = 1 n a i i per A ( i | i ) .
n 2 per A t r ( [ a i j per A ( i | j ) ] ) .
n 2 per A Sum of eigenvalues of   [ a i j per A ( i | j ) ] .
Theorem 3.
Let A Ω n and P = ( per A ( i / j ) ) = ( p i j ) . If kth row of P gives the max i j = 1 n p i j and m i n { a k j } = 1 n then
n per A min 1 i n j = 1 n per A ( i | j ) .
Proof. 
min i j = 1 n p i j max i j = 1 n p i j = j = 1 n p k j n j = 1 n a k j p k j = n per A , since a k j 1 n .
Example 1.
A = 1 3 1 3 1 3 1 3 1 3 1 3 1 3 1 3 1 3 6 7 1 7 1 7 6 7 , where is the direct sum.
It is easy to see that P = 74 441 74 441 74 441 74 441 74 441 74 441 74 441 74 441 74 441 4 21 2 63 2 63 4 21 . Maximum row sum of P = 74 147 and the minimum element of the row corresponding to the maximum row sum of P = 1 3 > 1 5 . Therefore, 5 p e r A min 1 i 5 j = 1 5 p e r A ( i | j ) .

3. Foregger’s Inequality

Let J n denote the n × n matrix with each entry equal to 1 n . Several authors have considered the problem of finding an upper bound for the permanent of a convex combination of J n and S, where S Ω n . Lih and Wang [16] discussed convexity inequality on the permanent of doubly stochastic matrices. For example, Marcus and Minc conjectured [17] that if S Ω n , n 2 , then per ( n J n S n 1 ) per S , equality holds when n = 2 . If n 3 then inequality holds iff S = J n . They established in [17] that the conjecture is true for n = 2 , or if S is positive semi-definite symmetric, or if S is in a sufficiently small neighborhood of J n .
E.T.H.Wang conjectured [18] that per ( n J n + S n + 1 ) per S and proved the Marcus and Minc conjecture for n = 3 , with a revised statement of the case of equality.
Foregger [9] raised a question whether per ( t J n + ( 1 t ) S ) per S for 0 t n n 1 , and S Ω n . He proved in [9] that for n = 3 , per ( t J 3 + ( 1 t ) S per S for 0 t 3 2 for S Ω 3 with equality iff S = J 3 or t = 3 2 and S is (up to permutations of rows and columns) 1 2 ( I + P ) , where P is a full-cycle permutation matrix. In addition, he proved in [10] that if S Ω 4 has all its off-diagonal entries less than or equal to 9 20 and t 0 < t 4 3 , where t 0 is the unique real root of 106 t 3 418 t 2 + 465 t 100 then per ( t J 4 + ( 1 t ) S ) per S with equality iff S = J 4 . This Foregger inequality is not yet proved for n 5 .
Subramanian and Somasundaram [8] proved that if A Ω n , 2 k n and the polynomial r = 2 k r c r σ r ( A J n ) t r 2 has no root in ( 0 , 1 ) where c r = ( k r ) ! n k r n r k r 2 then σ k ( t A + ( 1 t ) J n ) σ k ( A ) for all t [ 0 , 1 ] . In this paper, we prove that for all S Ω 5 and all t such that 0.25 t 0.6248 , per ( t J 5 + ( 1 t ) S ) per S . The following theorem is from Ebelein (Theorem 1, [19]).
Theorem 4.
Let ϕ ( x 1 , x 2 , , x n ) be a real symmetric polynomial of degree at most one in each variable defined for 0 x i 1 and i = 1 n x i = γ (γ is a real constant), then the maximum and minimum of ϕ ( x ) on the set C = { x | i = 1 n x i = γ and for i = 1 , 2 , , n , x i [ α i , β i ] , where [ α i , β i ] is any closed interval contained in [ 0 , 1 ] } and is assumed at least among the points whose components which are not end points are all equal. Moreover, if the maximum or minimum is attained only in the interior of C then it is assumed uniquely at the point ( γ n , γ n , γ n ) .
Let x be an n-dimensional vector. Then the elementary symmetric function of x denoted by e r ( x ) is the sum of products of coordinates of x taken r at a time. Let x = ( x 1 , x 2 , , x n ) . Then e r ( x ) = e r ( x 1 , x 2 , , x n ) , r = 1 , 2 , , n .
Theorem 5.
Let S Ω 5 have all its off-diagonal entries less than or equal to 9 20 and 0.25 t 0.6248 . Then per ( t J 5 + ( 1 t ) S ) per S .
Proof. 
Let S ( t ) = t J 5 + ( 1 t ) S . Then by Eberlein and Mudholkar ([20], p. 393)
per S ( t ) = 9 + T 1 ( S ( t ) ) ( e 2 + e 3 e 4 + 2 e 5 ) ( x ) + T 2 ( S ( t ) ( e 2 e 3 + e 4 2 e 5 ) ( x ) ,
where e r is the rth elementary symmetric function and T r ( B ) is the set of sums of columns of B, taken r at a time. If x T 1 ( S ( t ) ) then x = t e 5 + ( 1 t ) s where s T 1 ( S ) and e = [ 1 , 1 , 1 , 1 , 1 ] T . Hence,
e 2 ( x ) = 2 5 t 2 + t ( 1 t ) 4 5 + ( 1 t ) 2 e 2 ( s ) ,
e 3 ( x ) = 2 25 t 3 + 6 25 t 2 ( 1 t ) + 1 5 t ( 1 t ) 2 e 2 ( s ) + ( 1 t ) 3 e 3 ( s ) ,
e 4 ( x ) = 1 125 t 4 + 4 125 t 3 ( 1 t ) + 1 25 t 2 ( 1 t ) 2 e 2 ( s ) + 1 5 t ( 1 t ) 3 e 3 ( S ) + ( 1 t ) 4 e 4 ( s ) ,
e 5 ( x ) = 1 3125 t 5 + 4 625 t 4 ( 1 t ) + 1 125 t 3 ( 1 t ) 2 e 2 ( s ) + 1 25 t 2 ( 1 t ) 3 e 3 ( s ) + 1 5 t ( 1 t ) 4 e 4 ( s ) + ( 1 t ) 5 e 5 ( s ) .
Similarly if x T 2 ( S ( t ) ) then there exists r T 2 ( S ) such that x = 2 5 t e + ( 1 t ) r . Hence
e 2 ( x ) = 8 5 t 2 + 16 5 t ( 1 t ) + ( 1 t ) 2 e 2 ( r ) ,
e 3 ( x ) = 16 25 t 3 + 48 25 t 2 ( 1 t ) + 2 5 t ( 1 t ) 2 e 2 ( r ) + ( 1 t ) 3 e 3 ( r ) ,
e 4 ( x ) = 16 125 t 4 + 64 125 t 3 ( 1 t ) + 4 25 t 2 ( 1 t ) 2 e 2 ( r ) + 2 5 t ( 1 t ) 3 e 3 ( r ) + ( 1 t ) 4 e 4 ( r ) ,
e 5 ( x ) = 32 3125 t 5 + 128 625 t 4 ( 1 t ) + 8 125 t 3 ( 1 t ) 2 e 2 ( r ) + 4 25 t 2 ( 1 t ) 3 e 3 ( r ) + 2 5 t ( 1 t ) 4 e 4 ( r )
+ ( 1 t ) 5 e 5 ( r ) .
After substitution and simplification we have
per S ( t ) = per S + p 1 ( t ) + T 1 ( S ) ( p 2 ( t ) e 2 + p 3 ( t ) e 3 + p 4 ( t ) e 4 + p 5 ( t ) e 5 ) ( x ) +
T 2 ( s ) ( p 6 ( t ) e 2 + p 7 ( t ) e 3 + p 8 ( t ) e 4 + p 9 ( t ) e 5 ) ( x ) ,
where
p 1 ( t ) = 7246 3125 t 5 2901 625 t 4 + 1426 125 t 3 448 25 t 2 + 76 5 t ,
p 2 ( t ) = 2 125 t 5 9 125 t 4 + 37 125 t 3 36 25 t 2 + 11 5 t ,
p 3 ( t ) = 2 25 t 5 + 11 25 t 4 46 25 t 3 + 92 25 t 2 16 5 t
p 4 ( t ) = 2 5 t 5 13 5 t 4 + 32 5 t 3 22 5 t 2 + 22 5 t ,
p 5 ( t ) = 2 t 5 + 10 t 4 20 t 3 + 20 t 2 10 t ,
p 6 ( t ) = 16 125 t 5 + 52 125 t 4 56 125 t 3 + 49 25 t 2 12 5 t ,
p 7 ( t ) = 8 25 t 5 34 25 t 4 + 79 25 t 3 113 25 t 2 + 17 5 t
p 8 ( t ) = 4 5 t 5 + 21 5 t 4 44 5 t 3 + 46 5 t 2 24 5 t ,
p 9 ( t ) = 2 t 5 10 t 4 + 20 t 3 20 t 2 + 10 t .
Now use the identities ([20], p. 391)
T 2 ( A ) e 2 ( x ) = 3 T 1 ( A ) e 2 ( x ) + 10 and T 2 ( A ) e 3 ( x ) = T 1 ( A ) e 3 ( x ) + 3 T 1 ( A ) e 2 ( x )
to write
per S ( t ) = per S + 10 3 α + p 1 ( t ) + T 1 ( S ) ( p 2 + α + β + 3 γ ) e 2 + ( p 3 + γ + β 3 ) e 3 + p 4 ( t ) e 4 + p 5 ( t ) e 5 ) +
T 2 ( S ) ( ( p 6 ( t ) α 3 ) e 2 + ( p 7 ( t ) β 3 γ ) e 3 ) + p 8 ( t ) e 4 + p 9 ( t ) e 5 ) )
for any polynomials α , β and γ .
per S ( t ) = per S + c ( t ) + T 1 ( S ) f t ( s ) + T 2 ( S ) g t ( r ) ,
where f t = p 5 ( t ) e 5 + p 4 ( t ) e 4 + ( p 3 + γ + β / 3 ) e 3 + ( p 2 + α + β + 3 γ ) e 2 , g t = ( p 6 ( t ) α / 3 ) e 2 + ( p 7 ( t ) β / 3 γ ) e 3 + p 8 ( t ) e 4 + p 9 ( t ) e 5 ) and c ( t ) = 10 / 3 α + p 1 ( t ) .
We assume that all vectors in T 1 satisfy the condition 0 x 1 1 , 0 x i 9 20 , i = 2 , 3 , 4 , 5 . The functions f t and g t are linear combinations of elementary symmetric functions.
From Theorem 4, possible points of maximum of f t are [ 1 5 , 1 5 , 1 5 , 1 5 , 1 5 ] , [ 11 20 , 9 20 , 0 , 0 , 0 ] , [ 11 40 . 11 40 , 9 20 , 0 , 0 ] , [ 11 60 , 11 60 , 11 60 9 20 , 0 ] , [ 11 80 , 11 80 , 11 80 , 11 80 , 9 20 ] , [ 1 4 , 1 4 , 1 4 , 1 4 , 0 ] , [ 1 3 , 1 3 , 1 3 , 0 , 0 ] , [ 1 , 0 , 0 , 0 , 0 ] , [ 1 10 , 9 20 , 9 20 , 0 , 0 , ] , [ 1 20 . 9 20 , 9 20 , 1 20 , 0 ] , [ 1 30 , 9 20 , 9 20 , 1 30 , 1 30 ] and possible points of maximum of g t are [ 2 5 , 2 5 , 2 5 , 2 5 , 2 5 ] , [ 1 2 , 1 2 , 1 2 , 1 2 , 0 ] , [ 2 3 , 2 3 , 2 3 , 0 , 0 ] ,   [ 1 , 1 , 0 , 0 , 0 ] , [ 1 , 1 3 , 1 3 , 1 3 , 0 ] , [ 1 , 1 4 , 1 4 , 1 4 , 1 4 ] , [ 1 , 1 2 , 1 2 , 0 , 0 ] , [ 1 15 , 9 10 , 9 10 , 1 15 , 1 15 ] .
For each t, a set of linear inequalities must be satisfied in order for ( 1 5 , 1 5 , 1 5 , 1 5 , 1 5 ) to be a maximum for f t and for ( 2 5 , 2 5 , 2 5 , 2 5 , 2 5 ) to be a maximum for g t . These inequalities are solved numerically for various values of t and then interpolated to find α and β (details are shown in Appendix A). Substituting the values of α and β we obtain the values for f t ( s ) and g t ( s ) at different points. We have shown the values of f t ( s ) for different values of s and g t ( r ) for different values of r are given in the next two tables, respectively.
s f t ( s )
[ 1 5 , 1 5 , 1 5 , 1 5 , 1 5 ] 8 t 4 / 3125 7 t 4 / 625 + 29 / 12500 t 3 194 / 625 t 2 + 75397 / 187500 t 2609 / 375000
[ 11 20 . 9 20 , 0 , 0 , 0 ] 0.00396 t 5 0.01782 t 4 + 0.0647955 t 3 0.3564 t 2 + 0.38741175 t 0.0043065
[ 11 40 , 11 40 , 9 20 , 0 , 0 ] 979 / 400000 t 5 6633 / 800000 t 4 + 175813 / 8000000 t 3 68013 / 20000 t 2 + 127003657 / 320000000 t 1798797 / 320000000
[ 11 60 , 11 60 , 11 60 , 9 20 , 0 ] 201 / 78125 t 5 6061 / 625000 t 4 + 717247 / 50000000 t 3 5068 / 15625 t 2 + 58906841 / 150000000 t 1817611 / 300000000
[ 11 80 , 11 80 , 11 80 , 11 80 , 9 20 ] 0.0026 t 5 0.0104 t 4 + 0.0095 t 3 0.311744 t 2 + 0.3884 t 0.0063
[ 1 4 , 1 4 , 1 4 , 1 4 , 0 ] 8 t 5 / 3125 241 / 25000 t 4 + 1627 / 200000 t 3 8179 / 25000 t 2 + 969949 / 2400000 t 3131 / 480000
[ 1 3 , 1 3 , 1 3 , 0 , 0 ] 8 / 3375 t 5 26 / 3375 t 4 + 2581 / 135000 t 3 232 / 675 t 2 + 32663 / 81000 t 4697 / 810000
[ 1 2 , 1 2 , 0 , 0 , 0 ] 1 / 250 t 5 9 / 500 t 4 + 1309 / 20000 t 3 9 / 25 t 2 + 15653 / 40000 t 87 / 20000
[ 1 , 0 , 0 , 0 , 0 ] 0
[ 1 10 , 9 20 , 9 20 , 0 , 0 ] 191 / 62500 t 5 379 / 31250 t 4 + 78449 / 2000000 t 3 5414 / 15625 t 2 + 14733359 t / 37500000 1526647 / 300000000
[ 1 20 , 9 20 , 9 20 , 1 20 , 0 ] 197 / 25000 t 5 + 1953 / 50000 t 4 275569 / 1000000 t 3 + 2349 / 5000 t 2 606491 / 2000000 t 1281 / 250000
[ 1 30 , 9 20 , 9 20 , 1 30 , 1 30 ] 15669 / 5000000 t 5 63953 / 5000000 t 4 + 972461 / 25000000 t 3 171569 / 500000 t 2 + 117437989 / 300000000 t 1543843 / 300000000
r g t ( r )
[ 2 5 , 2 5 , 2 5 , 2 5 , 2 5 ] 256 3125 t 5 + 144 625 t 4 + 1257 3125 t 3 + 152 125 t 2 86134 46875 t + 291 31250
[ 1 2 , 1 2 , 1 2 , 1 2 , 0 ] 41 500 t 5 + 413 2000 t 4 + 3751 10000 t 3 + 251 200 t 2 112961 60000 t + 131 15000
[ 2 3 , 2 3 , 2 3 , 0 , 0 ] 11851 156250 t 5 + 94803 625000 t 4 + 17709461 50000000 t 3 + 159249 125000 t 2 191045273 100000000 t + 291289 37500000
[ 1 , 1 , 0 , 0 , 0 ] 16 125 t 5 + 52 125 t 4 2183 5000 t 3 + 49 25 t 2 32827 15000 t + 7 1200
[ 1 , 1 3 , 1 3 , 1 3 , 0 ] 12771 156250 t 5 + 128943 625000 t 4 + 13137261 50000000 t 3 + 63973 50000 t 2 91805513 50000000 t + 2329571 300000000
[ 1 , 1 4 , 1 4 , 1 4 , 1 4 ] 2033 25000 t 5 + 2711 12500 t 4 + 55171 200000 t 3 + 62519 50000 t 2 4323233 2400000 t + 1281 160000
[ 1 , 1 2 , 1 2 , 0 , 0 ] 2 25 t 5 + 9 50 t 4 + 977 4000 t 3 + 33 25 t 2 75423 40000 t + 437 60000
[ 1 15 , 9 10 , 9 10 , 1 15 , 1 15 ] 31377 312500 t 5 + 354931 1250000 t 4 1128039 50000000 t 3 + 49261 31250 t 2 602407919 300000000 t + 129307 18750000
In calculating the elementary symmetric functions and f t ( s ) and g t ( r ) at different points, MATLAB programs were used.
In the Appendix A, we have shown the curves f t ( s ) and g t ( r ) in Figure A1 and Figure A2, respectively. From the figures, f t ( s ) f t ( 1 5 , 1 5 , 1 5 , 1 5 , 1 5 ) in ( 0.25 , 0.98 ) . Furthermore, g t ( r ) g t ( 1 , 1 4 , 1 4 , 1 4 , 1 4 ) in ( 0.1 , 0.65 ) and g t ( r ) g t ( 2 5 , 2 5 , 2 5 , 2 5 , 2 5 ) in ( 0.65 , 1 ) . Therefore, in the interval (0.25, 0.65),
per S ( t ) T 2 ( S ) g t ( 1 , 1 4 , 1 4 , 1 4 , 1 4 ) + T 1 ( S ) f t ( 1 5 , 1 5 , 1 5 , 1 5 , 1 5 ) + per S + c ( t ) .
Similarly, in the interval (0.65, 0.98), we have,
p e r S ( t ) T 2 ( S ) g t ( 2 5 , 2 5 , 2 5 , 2 5 , 2 5 ) + T 1 ( S ) f t ( 1 5 , 1 5 , 1 5 , 1 5 , 1 5 ) + p e r S + c ( t ) .
Substituting the values, we obtain per S ( t ) per S in ( 0.25 , 0.6248 ) .
Holens [11] and Dokovic [12] proposed a conjecture (Holen–Dokovic conjecture), which states that if A Ω n , A J n and k is an integer, 1 k n , then σ k ( A ) ( n k + 1 ) 2 n k σ k 1 ( A ) . Dokovic proved that the conjecture is true for k 3 . Kopotun [21] proved that the conjecture is true for k = 4 and n 5 . Wanless [14] disproved this conjecture by providing a counterexample of order 22. The smallest order of a counterexample has not been established. In Theorem 6, we prove that the Holen–Dokovic conjecture fails for n = k = 4 . Before that, we recall that Foregger [10] proved that if A Ω 4 has all its off-diagonal entries less than or equal to 9 20 and t 0 < t 4 3 , where t 0 is the unique real root of 106 t 3 418 t 2 + 465 t 100 , then per ( t J 4 + ( 1 t ) A ) per S with equality if and only if A = J 4 .
Theorem 6.
The Holen–Dokovic conjecture fails for n = k = 4 .
Proof. 
Let Δ ( t ) = per A per ( t J 4 + ( 1 t ) A ) .
Foregger [10] proved that Δ ( t ) 0 in [ t 0 , 4 3 ] and t 0 is the unique real root of 106 t 3 418 t 2 + 465 t 100 .
Now, Δ ( t ) = per A r = 0 4 c r ( 1 t ) 4 r t r σ r ( A ) , where c r = ( n r ) ! n n r .
Here, Δ ( 1 ) = 0 and
Δ ( 1 ) = r = 0 4 c r σ r ( A ) [ ( 1 t ) 4 r r t r 1 + t r ( 1 t ) 3 r ( 1 ) ] t = 1 = 4 c 4 per A + c 3 σ 3 ( A ) = 4 per A + 1 4 σ 3 ( A )
Δ ( 1 ) = 4 per A + 1 4 σ 3 ( A )
Δ ( 1 ) = lim t 1 Δ ( t ) t 1 Δ ( 1 ) 0 iff Δ ( t ) 0 for all t [ 1 ϵ , 1 ] and Δ ( t ) 0 for all t [ 1 , 1 + ϵ ] , which is not the case since Δ ( t ) 0 for all t [ t 0 , 4 3 ] .
Hence, for some A Ω 4 , σ k ( A ) < ( n k + 1 ) 2 n k σ k 1 ( A ) . □

4. Conclusions

The Merris conjecture is one of the well-known conjectures in linear algebra and it is still open for n 4 . We proved the Merris inequality for all n × n non-negative matrices with minimum entry greater than or equal to 1 n . Furthermore, we gave a sufficient condition for a doubly stochastic matrix A to satisfy the Merris conjecture. Secondly, we proved the Foregger’s inequality. That is, for all S Ω 5 with off-diagonal entries less than or equal to 9 20 and all t such that 0.25 t 0.6248 , per ( t J 5 + ( 1 t ) S ) per S . Finally, we proved that the Holen–Dokovic conjecture fails for n = k = 4 and thus established that the smallest order of a counterexample to the Holen–Dokovic conjecture is n = 4 .

Author Contributions

K.S. and D.K.U. conceived and designed the experiments, performed the experiments, analyzed the data, contributed reagents/materials/analysis tools and wrote the paper. Both authors have read and agreed to the published version of the manuscript.

Funding

This research received no external funding.

Institutional Review Board Statement

Not applicable.

Informed Consent Statement

Not applicable.

Data Availability Statement

Not applicable.

Acknowledgments

The authors wish to thank the anonymous reviewers for their valuable comments.

Conflicts of Interest

The authors declare no conflict of interest.

Appendix A

In this Appendix, we have shown various calculations of α , β and γ values.
The inequalities f t ( 1 5 , 1 5 , 1 5 , 1 5 , 1 5 ) f t ( s ) and g t ( 2 5 , 2 5 , 2 5 , 2 5 , 2 5 ) g t ( r ) at t = 1
61 400 α + 43 240 β + 43 80 γ 0.10546
123 1600 α + 8851 96000 β + 8851 32000 γ 0.06386625
31 600 α + 0.06115432099 β + 0.183462963 γ 0.0678099537
5 128 α + 139 3072 β + 139 1024 γ 0.03263519287
1 40 α + 37 1200 β + 37 400 γ 0.02405375
1 15 α + 164 2025 β + 164 675 γ 0.0566637037
3 20 α + 53 300 β + 53 100 γ 0.10296
10 25 α + 32 75 β + 32 25 γ 0.35296
43 400 α + 1529 12000 β + 1529 4000 γ 0.08071
21 200 α + 149 1200 β + 149 400 γ 0.08046
5 48 α + 319 2592 β + 319 864 γ 0.0775237037
4.166666667 3 α 0.2002037037 3 β 0.2002037037 γ 0.1736953067
1 30 α 7 150 β 7 50 γ 0.01284375
4 45 α 232 2025 β 232 675 γ 0.004136296296
1 5 α 16 75 β 16 25 γ 0.01984 .
1 120 α 13 1200 β 13 400 γ 0.00070984375 .
0.1389 α 0.1512 β 0.4537 γ 0.1070
If γ = s , β = 3 s , then α 0.6864 .
We can take α = 0.6864 , γ = 1 , β = 3 .
The inequalities f t ( 1 5 , 1 5 , 1 5 , 1 5 , 1 5 ) f t ( s ) and g t ( 2 5 , 2 5 , 2 5 , 2 5 , 2 5 ) g t ( r ) at t = 1 2
61 400 α + 43 240 β + 43 80 γ 0.0604625
123 1600 α + 8851 96000 β + 8851 32000 γ 0.03212203125
31 600 α + 0.06115432099 β + 0.183462963 γ 0.01752152778
5 128 α + 139 3072 β + 139 1024 γ 0.01331022355
1 40 α + 37 1200 β + 37 400 γ 0.0103815625
1 15 α + 164 2025 β + 164 675 γ 0.02689111111
3 20 α + 53 300 β + 53 100 γ 0.05853
2 5 α + 32 75 β + 32 25 γ 0.25178
43 400 α + 1529 12000 β + 1529 4000 γ 0.04359875
21 200 α + 149 1200 β + 149 400 γ 0.0427715625
5 48 α + 319 2592 β + 319 864 γ 0.04248444444
0.00138888889 α 0.0667345679 β 0.2002037037 γ 0.1962603704
1 30 α 7 150 β 7 50 γ 0.03498
4 45 α 232 2025 β 232 675 γ 0.10752
1 5 α 16 75 β 16 25 γ 0.17248
1 120 α 13 1200 β 13 400 γ 0.00748375
0.1389 α 0.1512 β 0.4537 γ 0.2793
If γ = s , β = 3 s , then α 0.3391263441 .
We can take α = 0.3391263441 , γ = 2 , β = 6
The inequalities f t ( 1 5 , 1 5 , 1 5 , 1 5 , 1 5 ) f t ( s ) and g t ( 2 5 , 2 5 , 2 5 , 2 5 , 2 5 ) g t ( r ) at t = 1 4 .
61 400 α + 43 240 β + 43 80 γ 0.02987980469
123 1600 α + 8851 96000 β + 8851 32000 γ 0.01508300537
209 600 α + 0.3655123457 β + 1.096537037 γ 0.1335164742
5 128 α + 139 3072 β + 139 1024 γ 0.009529170096
1 40 α + 37 1200 β + 37 400 γ 0.004418186035
1 15 α + 164 2025 β + 164 675 γ 0.0121374537
3 20 α + 53 300 β + 53 100 γ 0.02871890625
2 5 α + 32 75 β + 32 25 γ 0.14480875
43 400 α + 1529 12000 β + 1529 4000 γ 0.02107509766
21 200 α + 149 1200 β + 149 400 γ 0.02128248291
8187 21600 α + 0.3979320988 β + 1.193796296 γ 0.1489750101
0.001388888889 α 0.2002037037 3 β 0.2002037037 γ 0.1169795821
1 30 α 7 150 β 7 50 γ 0.1006621875
4 45 α 232 2025 β 232 675 γ 0.2591237037
1 5 α 16 75 β 16 25 γ 0.60142
1 120 α 13 1200 β 13 400 γ 0.02454455811
0.1389 α 0.1512 β 0.4537 γ 0.0044
If γ = s , β = 3 s , then α 0.1767274414 .
We can take α = 0.1767274414 , γ = 3 , β = 9 .
Interpolating the values of α , β , γ at t = 1 , 1 2 , 1 4 we obtain
α = 0.0342 t 3 0.6346 t 0.0175
γ = 1.5238 t 3 4.6667 t + 4.1429
β = 4.5714 t 3 + 14 t 12.4286 .
Figure A1 and Figure A2 are showing f t ( s ) and g t ( r ) for different values of s and r.
Figure A1. Curves f t ( s ) .
Figure A1. Curves f t ( s ) .
Symmetry 13 01782 g0a1
Figure A2. Curves g t ( r ) .
Figure A2. Curves g t ( r ) .
Symmetry 13 01782 g0a2

References

  1. Minc, H. Permanents. Encycl. Math. Its Appl. 1978, 6, 158. [Google Scholar]
  2. Lieb, E.H. Proofs of some conjectures on permanents. In Inequalities; Springer: Berlin/Heidelberg, Germany, 2002; pp. 101–108. [Google Scholar]
  3. Hwang, S.G. A note on a conjecture on permanents. Linear Algebra Its Appl. 1986, 76, 31–44. [Google Scholar] [CrossRef] [Green Version]
  4. Lih, K.W.; Wang, E.T. Monotonicity conjecture on permanents of doubly stochastic matrices. Proc. Am. Math. Soc. 1981, 82, 173–178. [Google Scholar] [CrossRef]
  5. Cheon, G.S.; Wanless, I.M. An update on Minc’s survey of open problems involving permanents. Linear Algebra Its Appl. 2005, 403, 314–342. [Google Scholar] [CrossRef] [Green Version]
  6. Zhang, F. An update on a few permanent conjectures. Spec. Matrices 2016, 4, 305–316. [Google Scholar] [CrossRef] [Green Version]
  7. Merris, R. The permanent of a doubly stochastic matrix. Am. Math. Mon. 1973, 80, 791–793. [Google Scholar] [CrossRef]
  8. Subramanian, P.; Somasundaram, K. Some conjectures on permanents of doubly stochastic matrices. J. Discret. Math. Sci. Cryptogr. 2016, 19, 997–1011. [Google Scholar] [CrossRef]
  9. Foregger, T.H. Remarks on a Conjecture of M. Marcus and H. Minc. Linear Multilinear Algebra 1979, 7, 123–126. [Google Scholar] [CrossRef]
  10. Foregger, T.H. Permanents of convex combinations of doubly stochastic matrices. Linear Multilinear Algebra 1988, 23, 79–90. [Google Scholar] [CrossRef]
  11. Holens, F. Two Aspects of Doubly Stochastic Matrices: Permutation Matrices and the Minimum Permanent Function. Ph.D. Thesis, University of Manitoba, Winnipeg, MB, Canada, 1964. [Google Scholar]
  12. Djokovic, D.Z. On a conjecture by van der Waerden. Mat. Vesnik 1967, 19, 566–569. [Google Scholar]
  13. Hwang, S.G. The monotonicity of and the Đoković conjectures on permanents of doubly stochastic matrices. Linear Algebra Its Appl. 1986, 79, 127–151. [Google Scholar] [CrossRef] [Green Version]
  14. Wanless, I.M. The Holen-Dokovic conjecture on permanents fails. Linear Algebra Its Appl. 1999, 286, 273–285. [Google Scholar] [CrossRef] [Green Version]
  15. Seneta, E. Non-Negative Matrices and Markov Chains; Springer: New York, NY, USA, 1981. [Google Scholar]
  16. Lih, K.W.; Wang, E.T. A convexity inequality on the permanent of doubly stochastic matrices. Congr. Numer. 1982, 36, 189–198. [Google Scholar]
  17. Marcus, M.; Minc, H. On a conjecture of B. L. Van der Waerden. Math. Proc. Camb. Philos. Soc. 1967, 63, 305–309. [Google Scholar] [CrossRef]
  18. Wang, E.T. On a conjecture of M. Marcus and H. Minc. Linear Multilinear Algebra 1977, 5, 145–148. [Google Scholar] [CrossRef]
  19. Eberlein, P.J. Remarks on the van der Waerden conjecture II. Linear Algebra Its Appl. 1969, 2, 311–320. [Google Scholar] [CrossRef] [Green Version]
  20. Eberlein, P.J.; Mudholkar, G.S. Some remarks on the van der Waerden conjecture. J. Comb. Theory 1968, 5, 386–396. [Google Scholar] [CrossRef] [Green Version]
  21. Kopotun, K.A. On some permanental conjectures. Linear Multilinear Algebra 1994, 36, 205–216. [Google Scholar] [CrossRef]
Publisher’s Note: MDPI stays neutral with regard to jurisdictional claims in published maps and institutional affiliations.

Share and Cite

MDPI and ACS Style

Udayan, D.K.; Somasundaram, K. The Inequalities of Merris and Foregger for Permanents. Symmetry 2021, 13, 1782. https://doi.org/10.3390/sym13101782

AMA Style

Udayan DK, Somasundaram K. The Inequalities of Merris and Foregger for Permanents. Symmetry. 2021; 13(10):1782. https://doi.org/10.3390/sym13101782

Chicago/Turabian Style

Udayan, Divya K., and Kanagasabapathi Somasundaram. 2021. "The Inequalities of Merris and Foregger for Permanents" Symmetry 13, no. 10: 1782. https://doi.org/10.3390/sym13101782

Note that from the first issue of 2016, this journal uses article numbers instead of page numbers. See further details here.

Article Metrics

Back to TopTop